Difference between revisions of "2002 AIME II Problems/Problem 15"

m
(Solution)
Line 2: Line 2:
 
Circles <math>\mathcal{C}_{1}</math> and <math>\mathcal{C}_{2}</math> intersect at two points, one of which is <math>(9,6)</math>, and the product of the radii is <math>68</math>. The x-axis and the line <math>y = mx</math>, where <math>m > 0</math>, are tangent to both circles. It is given that <math>m</math> can be written in the form <math>a\sqrt {b}/c</math>, where <math>a</math>, <math>b</math>, and <math>c</math> are positive integers, <math>b</math> is not divisible by the square of any prime, and <math>a</math> and <math>c</math> are relatively prime. Find <math>a + b + c</math>.
 
Circles <math>\mathcal{C}_{1}</math> and <math>\mathcal{C}_{2}</math> intersect at two points, one of which is <math>(9,6)</math>, and the product of the radii is <math>68</math>. The x-axis and the line <math>y = mx</math>, where <math>m > 0</math>, are tangent to both circles. It is given that <math>m</math> can be written in the form <math>a\sqrt {b}/c</math>, where <math>a</math>, <math>b</math>, and <math>c</math> are positive integers, <math>b</math> is not divisible by the square of any prime, and <math>a</math> and <math>c</math> are relatively prime. Find <math>a + b + c</math>.
  
== Solution ==
+
== Solution 1==
 
Let the smaller angle between the <math>x</math>-axis and the line <math>y=mx</math> be <math>\theta</math>. Note that the centers of the two circles lie on the angle bisector of the angle between the <math>x</math>-axis and the line <math>y=mx</math>. Also note that if <math>(x,y)</math> is on said angle bisector, we have that <math>\frac{y}{x}=\tan{\frac{\theta}{2}}</math>.  Let <math>\tan{\frac{\theta}{2}}=m_1</math>, for convenience. Therefore if <math>(x,y)</math> is on the angle bisector, then <math>x=\frac{y}{m_1}</math>. Now let the centers of the two relevant circles be <math>(a/m_1 , a)</math> and <math>(b/m_1 , b)</math> for some positive reals <math>a</math> and <math>b</math>. These two circles are tangent to the <math>x</math>-axis, so the radii of the circles are <math>a</math> and <math>b</math> respectively. We know that the point <math>(9,6)</math> is a point on both circles, so we have that
 
Let the smaller angle between the <math>x</math>-axis and the line <math>y=mx</math> be <math>\theta</math>. Note that the centers of the two circles lie on the angle bisector of the angle between the <math>x</math>-axis and the line <math>y=mx</math>. Also note that if <math>(x,y)</math> is on said angle bisector, we have that <math>\frac{y}{x}=\tan{\frac{\theta}{2}}</math>.  Let <math>\tan{\frac{\theta}{2}}=m_1</math>, for convenience. Therefore if <math>(x,y)</math> is on the angle bisector, then <math>x=\frac{y}{m_1}</math>. Now let the centers of the two relevant circles be <math>(a/m_1 , a)</math> and <math>(b/m_1 , b)</math> for some positive reals <math>a</math> and <math>b</math>. These two circles are tangent to the <math>x</math>-axis, so the radii of the circles are <math>a</math> and <math>b</math> respectively. We know that the point <math>(9,6)</math> is a point on both circles, so we have that
  
Line 30: Line 30:
  
 
It then follows that <math>m=\tan{\theta}=\frac{12\sqrt{221}}{49}</math>. Therefore <math>a=12</math>, <math>b=221</math>, and <math>c=49</math>. The desired answer is then <math>12+221+49=\boxed{282}</math>.
 
It then follows that <math>m=\tan{\theta}=\frac{12\sqrt{221}}{49}</math>. Therefore <math>a=12</math>, <math>b=221</math>, and <math>c=49</math>. The desired answer is then <math>12+221+49=\boxed{282}</math>.
 +
 +
== Solution 2 (Alcumus)==
 +
Let <math>r_1</math> and <math>r_2</math> be the radii of the circles. Then the centers of the circles are of the form <math>(kr_1,r_1)</math> and <math>(kr_2,r_2)</math> for the same constant <math>k,</math> since the two centers are collinear with the origin. Since <math>(9,6)</math> lies on both circles,
 +
<cmath>(kr - 9)^2 + (r - 6)^2 = r^2,</cmath>where <math>r</math> represents either radius. Expanding, we get
 +
<cmath>k^2 r^2 - (18k + 12) r + 117 = 0.</cmath>We are told the product of the circles is 68, so by Vieta's formulas, <math>\frac{117}{k^2} = 68.</math> Hence, <math>k^2 = \frac{117}{68},</math> and <math>k = \sqrt{\frac{117}{68}}.</math>
 +
 +
[asy]
 +
unitsize(0.25 cm);
 +
 +
pair[] O;
 +
real[] r;
 +
pair P;
 +
 +
r[1] = 4.096;
 +
r[2] = 16.6;
 +
O[1] = (r[1]/(2/3*sqrt(17/13)),r[1]);
 +
O[2] = (r[2]/(2/3*sqrt(17/13)),r[2]);
 +
P = reflect(O[1],O[2])*(9,6);
 +
 +
draw(Circle(O[1],r[1]));
 +
//draw(Circle(O[2],r[2]));
 +
draw(arc(O[2],r[2],130,300));
 +
draw((0,0)--(8,12*sqrt(221)/49*8));
 +
draw((0,0)--(30,0));
 +
draw((0,0)--O[1]--(O[1].x,0));
 +
draw(O[1]--(O[1] + reflect((0,0),(10,12*sqrt(221)/49*10))*(O[1]))/2);
 +
 +
label("<math>y = mx</math>", (8,12*sqrt(221)/49*8), N);
 +
 +
dot("<math>(9,6)</math>", (9,6), NE);
 +
dot("<math>(kr,r)</math>", O[1], N);
 +
dot(P,red);
 +
[/asy]
 +
 +
Since the circle is tangent to the line <math>y = mx,</math> the distance from the center <math>(kr,r)</math> to the line is <math>r.</math> We can write <math>y = mx</math> as <math>y - mx = 0,</math> so from the distance formula,
 +
<cmath>\frac{|r - krm|}{\sqrt{1 + m^2}} = r.</cmath>Squaring both sides, we get
 +
<cmath>\frac{(r - krm)^2}{1 + m^2} = r^2,</cmath>so <math>(r - krm)^2 = r^2 (1 + m^2).</math> Since <math>r \neq 0,</math> we can divide both sides by 0, to get
 +
<cmath>(1 - km)^2 = 1 + m^2.</cmath>Then <math>1 - 2km + k^2 m^2 = 1 + m^2,</math> so <math>m^2 (1 - k^2) + 2km = 0.</math> Since <math>m \neq 0,</math>
 +
<cmath>m(1 - k^2) + 2k = 0.</cmath>Hence,
 +
<cmath>m = \frac{2k}{k^2 - 1} = \frac{2 \sqrt{\frac{117}{68}}}{\frac{117}{68} - 1} = \boxed{\frac{12 \sqrt{221}}{49}}.</cmath>
  
 
== See also ==
 
== See also ==

Revision as of 14:16, 1 March 2021

Problem

Circles $\mathcal{C}_{1}$ and $\mathcal{C}_{2}$ intersect at two points, one of which is $(9,6)$, and the product of the radii is $68$. The x-axis and the line $y = mx$, where $m > 0$, are tangent to both circles. It is given that $m$ can be written in the form $a\sqrt {b}/c$, where $a$, $b$, and $c$ are positive integers, $b$ is not divisible by the square of any prime, and $a$ and $c$ are relatively prime. Find $a + b + c$.

Solution 1

Let the smaller angle between the $x$-axis and the line $y=mx$ be $\theta$. Note that the centers of the two circles lie on the angle bisector of the angle between the $x$-axis and the line $y=mx$. Also note that if $(x,y)$ is on said angle bisector, we have that $\frac{y}{x}=\tan{\frac{\theta}{2}}$. Let $\tan{\frac{\theta}{2}}=m_1$, for convenience. Therefore if $(x,y)$ is on the angle bisector, then $x=\frac{y}{m_1}$. Now let the centers of the two relevant circles be $(a/m_1 , a)$ and $(b/m_1 , b)$ for some positive reals $a$ and $b$. These two circles are tangent to the $x$-axis, so the radii of the circles are $a$ and $b$ respectively. We know that the point $(9,6)$ is a point on both circles, so we have that

\[(9-\frac{a}{m_1})^2+(6-a)^2=a^2\]

\[(9-\frac{b}{m_1})^2+(6-b)^2=b^2\]

Expanding these and manipulating terms gives

\[\frac{1}{m_1^2}a^2-[(18/m_1)+12]a+117=0\]

\[\frac{1}{m_1^2}b^2-[(18/m_1)+12]b+117=0\]

It follows that $a$ and $b$ are the roots of the quadratic

\[\frac{1}{m_1^2}x^2-[(18/m_1)+12]x+117=0\]

It follows from Vieta's Formulas that the product of the roots of this quadratic is $117m_1^2$, but we were also given that the product of the radii was 68. Therefore $68=117m_1^2$, or $m_1^2=\frac{68}{117}$. Note that the half-angle formula for tangents is

\[\tan{\frac{\theta}{2}}=\sqrt{\frac{1-\cos{\theta}}{1+\cos{\theta}}}\]

Therefore

\[\frac{68}{117}=\frac{1-\cos{\theta}}{1+\cos{\theta}}\]

Solving for $\cos{\theta}$ gives that $\cos{\theta}=\frac{49}{185}$. It then follows that $\sin{\theta}=\sqrt{1-\cos^2{\theta}}=\frac{12\sqrt{221}}{185}$.

It then follows that $m=\tan{\theta}=\frac{12\sqrt{221}}{49}$. Therefore $a=12$, $b=221$, and $c=49$. The desired answer is then $12+221+49=\boxed{282}$.

Solution 2 (Alcumus)

Let $r_1$ and $r_2$ be the radii of the circles. Then the centers of the circles are of the form $(kr_1,r_1)$ and $(kr_2,r_2)$ for the same constant $k,$ since the two centers are collinear with the origin. Since $(9,6)$ lies on both circles, \[(kr - 9)^2 + (r - 6)^2 = r^2,\]where $r$ represents either radius. Expanding, we get \[k^2 r^2 - (18k + 12) r + 117 = 0.\]We are told the product of the circles is 68, so by Vieta's formulas, $\frac{117}{k^2} = 68.$ Hence, $k^2 = \frac{117}{68},$ and $k = \sqrt{\frac{117}{68}}.$

[asy] unitsize(0.25 cm);

pair[] O; real[] r; pair P;

r[1] = 4.096; r[2] = 16.6; O[1] = (r[1]/(2/3*sqrt(17/13)),r[1]); O[2] = (r[2]/(2/3*sqrt(17/13)),r[2]); P = reflect(O[1],O[2])*(9,6);

draw(Circle(O[1],r[1])); //draw(Circle(O[2],r[2])); draw(arc(O[2],r[2],130,300)); draw((0,0)--(8,12*sqrt(221)/49*8)); draw((0,0)--(30,0)); draw((0,0)--O[1]--(O[1].x,0)); draw(O[1]--(O[1] + reflect((0,0),(10,12*sqrt(221)/49*10))*(O[1]))/2);

label("$y = mx$", (8,12*sqrt(221)/49*8), N);

dot("$(9,6)$", (9,6), NE); dot("$(kr,r)$", O[1], N); dot(P,red); [/asy]

Since the circle is tangent to the line $y = mx,$ the distance from the center $(kr,r)$ to the line is $r.$ We can write $y = mx$ as $y - mx = 0,$ so from the distance formula, \[\frac{|r - krm|}{\sqrt{1 + m^2}} = r.\]Squaring both sides, we get \[\frac{(r - krm)^2}{1 + m^2} = r^2,\]so $(r - krm)^2 = r^2 (1 + m^2).$ Since $r \neq 0,$ we can divide both sides by 0, to get \[(1 - km)^2 = 1 + m^2.\]Then $1 - 2km + k^2 m^2 = 1 + m^2,$ so $m^2 (1 - k^2) + 2km = 0.$ Since $m \neq 0,$ \[m(1 - k^2) + 2k = 0.\]Hence, \[m = \frac{2k}{k^2 - 1} = \frac{2 \sqrt{\frac{117}{68}}}{\frac{117}{68} - 1} = \boxed{\frac{12 \sqrt{221}}{49}}.\]

See also

2002 AIME II (ProblemsAnswer KeyResources)
Preceded by
Problem 14
Followed by
Last Question
1 2 3 4 5 6 7 8 9 10 11 12 13 14 15
All AIME Problems and Solutions

The problems on this page are copyrighted by the Mathematical Association of America's American Mathematics Competitions. AMC logo.png